Đến nội dung

Pham Le Yen Nhi nội dung

Có 96 mục bởi Pham Le Yen Nhi (Tìm giới hạn từ 21-04-2020)



Sắp theo                Sắp xếp  

#392552 $\boxed{\text{Chuyên Đề}}$ Phân tích đa thức thành nhân tử

Đã gửi bởi Pham Le Yen Nhi on 02-02-2013 - 19:25 trong Đại số

Bài 4: Ta có: $ab+cd=ab.1+cd.1=ab.(c^{2}+d^{2})+cd.(a^{2}+b^{2})$
$\Rightarrow ab+cd=abc^{2}+abd^{2}+cda^{2}+cdb^{2}$$
\Rightarrow ab+cd=ad(bd+ac)+bc(ac+bd)$
$\Rightarrow ab+cd=(ac+bd)(bc+ad)$
mà $ac+bd=0$
Vậy $ab+cd=0$



#494081 [Violympic9] Các bài toán violympic lớp 9 cho kì thi quốc gia sắp tới.

Đã gửi bởi Pham Le Yen Nhi on 20-04-2014 - 11:01 trong Cuộc thi VIOlympic (Cuộc thi do Bộ giáo dục và đào tạo tổ chức)

2) Cho pt: $\frac{x^2}{(x-1)^2}+\frac{x^2}{(x+1)^2}=\frac{10}{9}$

Tập hợp nghiệm của pt là : ...

ĐK:$x\neq 1,-1$

Ta có phương trình $\Leftrightarrow 8x^{4}+38x^{2}-10=0$

đưa về phương trình trùng phương và giải ta được $x=\frac{1}{2}$ hoặc $x=\frac{-1}{2}$ (TMĐK)




#494090 [Violympic9] Các bài toán violympic lớp 9 cho kì thi quốc gia sắp tới.

Đã gửi bởi Pham Le Yen Nhi on 20-04-2014 - 11:11 trong Cuộc thi VIOlympic (Cuộc thi do Bộ giáo dục và đào tạo tổ chức)

Lưu ý: Trả lời cách làm chứ không phải đáp án
1) Cho tam giác $ABC$ vuông tại $A$. có $AB=5cm$ và $BC=10cm$. Gọi $M;N;P$ lần lượt là trung điểm của các cạnh $AB;BC;CA$. Chu vi đường tròn ngoại tiếp tam giác $MNP$ là ... $(cm)$ (Kết quả lẻ)

Dễ dàng chứng minh được $\Delta MNP$ vuông tại $N$ và $MP=\frac{BC}{2}=5$ (cm)

từ đó tính được chu vi đường tròn ngoại tiếp $\Delta MNP$ : $C= 2R\Pi =5\Pi$ (cm)




#381562 Chứng minh $\frac{x}{2b+2c-a}=\frac{y...

Đã gửi bởi Pham Le Yen Nhi on 29-12-2012 - 14:23 trong Đại số

Bài 7:
Ta có:
$\frac{x^{2}+y^{2}+z^{2}}{a^{2}+b^{2}+c^{2}}= \frac{x^{2}}{a^{2}}+\frac{y^{2}}{b^{2}}+\frac{z^{2}}{c^{2}}$
$\Rightarrow \frac{x^{2}+y^{2}+z^{2}}{a^{2}+b^{2}+c^{2}}-\frac{x^{x}}{a^{2}}-\frac{y^{2}}{b^{2}}-\frac{z^{2}}{c^{2}}=0$$
\Rightarrow \left ( \frac{x^{2}}{a^{2}+b^{2}+c^{2}} -\frac{x^{2}}{b^{2}}\right )+\left ( \frac{y^{2}}{a^{2}+b^{2}+c^{2}}-\frac{y^{2}}{b^{2}} \right )+\left ( \frac{z^{2}}{a^{2}+b^{2}+c^{2}}-\frac{z^{2}}{c^{2}} \right )=0$
$\Rightarrow x^{2}\left ( \frac{1}{a^{2}+b^{2}+c^{2}}-\frac{1}{a^{2}} \right )+y^{2}\left ( \frac{1}{a^{2}+b^{2}+c^{2}}-\frac{1}{b^{2}} \right )+z^{2}\left ( \frac{1}{a^{2}+b^{2}+c^{2}}-\frac{1}{c^{2}} \right )=0$
$\Rightarrow x^{2}\left ( \frac{-b^{2}-c^{2}}{a^{2}\left ( a^{2}+b^{2}+c^{2}\right )} \right )+y^{2}\left ( \frac{-a^{2}-c^{2}}{b^{2}\left ( a^{2}+b^{2}+c^{2}\right )} \right )+z^{2}\left ( \frac{-b^{2}-a^{2}}{c^{2}\left ( a^{2}+b^{2}+c^{2}\right )} \right )=0$
mà $a,b,c\neq 0$
nên $x^{2}=y^{2}=z^{2}=0$
Vậy x=y=z=0



#381344 Chứng minh $\frac{x}{2b+2c-a}=\frac{y...

Đã gửi bởi Pham Le Yen Nhi on 28-12-2012 - 21:03 trong Đại số

Bài 1:
Gọi d là ước chung của $n^{3}+2n$ và $n^{4}+3n^{2}+1$
Ta có:
$n^{3}+2n \vdots d\Rightarrow n\left ( n^{3} +2n\right )\vdots d \Rightarrow n^{4}+2n^{2}\vdots d \left ( 1 \right )$
Ta có:
$n^{4}+3n^{2}+1-n^{4}-2n^{2} \vdots d \Rightarrow n^{2}+1\vdots d
\Rightarrow \left ( n^{2}+1 \right )^{2}\vdots d
\Rightarrow n^{4}+2n^{2}+1\vdots d$$\left ( 2\right )$
Từ$\left ( 1\right )$ và $\left ( 2\right )$
$\Rightarrow \left ( n^{4}+2n^{2}+1 \right )-\left ( n^{4}+2n \right )\vdots d \Rightarrow 1\vdots d \Rightarrow d=\pm 1$
Vậy $\frac{n^{3}+2n}{n^{4}+2n+1}$ là phân thức tối giản với mọi $n \epsilon \mathbb {N}$
Bài này trong NC và phát triển toán 8 (còn 1 cách giải nữa)



#501905 Đề thi $TS$ lớp $10$ $THPT$ chuyên Thái Bình...

Đã gửi bởi Pham Le Yen Nhi on 27-05-2014 - 11:09 trong Tài liệu - Đề thi

Đề thi TS lớp 10 THPT chuyên TB 2010-2011 (V2)

Bài 2:
Cho hệ: $\left\{\begin{matrix}ax+by=c  &  & \\ bx+cy=a  &  & \\ cx+ay=b \end{matrix}\right.$ với $a;b;c$ là tham số.

Cmr: Điều kiện cần và đủ để hệ phương trình trên có nghiệm là: $a^3+b^3+c^3=3abc$

Giả sử $\left ( x_{o} ,y_{o}\right )$ là nghiệm của hệ phương trình đã cho

ta có hệ 

$\left\{\begin{matrix} ax_{o}+by_{o}=c (1)\\ bx_{o}+cy_{o}=a(2)\\ cx_{o}+ay_{o}=b (3) \end{matrix}\right.$

+)Nhân hai vế của $(1),(2),(3)$ lần lượt với $c^{2},a^{2},b^{2}$ ta dễ dàng suy ra được

$a^{3}+b^{3}+c^{3}=(a^{2}b+b^{2}c+c^{2}a)x_{o}+(a^{2}c+b^{2}a+c^{2}b)y_{o}$

+)Nhân hai vế của $(1),(2),(3)$ lần lượt với $ab,bc,ca$ ta suy ra được $3abc=(a^{2}b+b^{2}c+c^{2}a)x_{o}+(a^{2}c+b^{2}a+c^{2}b)y_{o}$

Từ đó suy ra được $a^{3}+b^{3}+c^{3}=3abc$




#503024 $x+2y\leq \frac{3+\sqrt{10}}{2...

Đã gửi bởi Pham Le Yen Nhi on 31-05-2014 - 18:29 trong Bất đẳng thức và cực trị

Bài 1: Cho $x^{2}+y^{2}\leq x+y$. Chứng minh rằng $x+2y\leq \frac{3+\sqrt{10}}{2}$

 

Bài 2: Cho $x, y$ là hai số tự nhiên khác $0$ thỏa mãn $2x+3y = 53$. Tìm giá trị lớn nhất của biểu thức $P=\sqrt{xy+4}$

 

Bài 3: Cho $x,y,z >0$, $x+y+z=1$. Tìm GTNN của $S=\frac{1}{x}+\frac{4}{y}+\frac{9}{z}$

Bài 3: ta có 

$\frac{1}{x}+\frac{4}{y}+\frac{9}{z}\geq \frac{(1+2+3)^{2}}{x+y+z}=36$

Dấu "=" xảy ra $\Leftrightarrow \left ( x,y,z \right )=\left ( \frac{1}{6};\frac{1}{3};\frac{1}{2} \right )$




#503031 $x+2y\leq \frac{3+\sqrt{10}}{2...

Đã gửi bởi Pham Le Yen Nhi on 31-05-2014 - 18:52 trong Bất đẳng thức và cực trị

Bài 2: Cho $x, y$ là hai số tự nhiên khác $0$ thỏa mãn $2x+3y = 53$. Tìm giá trị lớn nhất của biểu thức $P=\sqrt{xy+4}$

ta có $53=2x+3y\geq 2\sqrt{2x.3y}\Rightarrow xy\leq \frac{2809}{24}$

$\Rightarrow P\leq \frac{\sqrt{17430}}{12}$

Dấu "=" xảy ra $\Leftrightarrow x=\frac{53}{4},y=\frac{53}{6}$

p/s: @caovannct hình như bài bạn giải là bài 1 




#480242 $P=\sqrt{-x^{2}+4x+12}-\sqrt{-x^...

Đã gửi bởi Pham Le Yen Nhi on 01-02-2014 - 10:44 trong Đại số

ĐKXĐ :$-1\leq x\leq 3$ (1)

xét hiệu : $(-x^{2}+4x+12)-(-x^{2}+2x+3)=2x+9$

do (1) nên $2x+9 > 0$ do đó $P> 0$

Xét $P^{2}=(x+2)(6-x)+(x+1)(3-x)-2\sqrt{(x+2)(6-x)(x+2)(6-x)}$

              $=(x+1)(6-x)+(6-x)+(x+2)(3-x)-(3-x)-2\sqrt{(x+2)(6-x)(x+2)(6-x)}$

              $=(x+1)(6-x)+(x+2)(3-x)-2\sqrt{(x+2)(6-x)(x+2)(6-x)}+3$

              $=(\sqrt{(x+1)(6-x)}-\sqrt{(x+2)(3-x)})^{2}+3$

Do đó $P^{2}\geq 3 => P\geq \sqrt{3}$ (vì $P\geq 0$)

Vậy min $P=\sqrt{3}\Leftrightarrow x=0$




#489422 Đề thi học sinh giỏi lớp 9 tỉnh Đăk Lăk năm 2013-2014

Đã gửi bởi Pham Le Yen Nhi on 29-03-2014 - 17:42 trong Tài liệu - Đề thi

Câu 1.(4 điểm)

a) Chứng minh rằng $\sqrt[3]{9+4\sqrt{5}}+\sqrt[3]{9-4\sqrt{5}}$ là một số nguyên

b)Cho số n nguyên dương tùy ý .Xét ba số tự nhiên là $a=11...1$(có 2n chữ số 1),$b=11...1$(có n+1 chữ số 1) và $c=66...6$(có n chữ số 6).Chứng minh rằng $a+b+c+8$ là một số chính phương

Câu 2.(4 điểm)

a)Cho x,y là hai số thực thỏa mãn điều kiện $x^2+y^2\leq x+y$.Chứng minh rằng $x+y\leq 2$

b)Giải phương trình $x^2+\frac{x^2}{(x+1)^2}=15$

Câu 3.(4 điểm)

a)Giải hệ phương trình 

$\left\{\begin{matrix} (x+y+z)^2=3(xy+yz+xz)\\ x^{2013}+y^{2013}+z^{2013}=3^{2014} \end{matrix}\right.$

b)Tìm tất cả các cặp số nguyên (x,y) thỏa mãn phương trình $x^3-y^3+2x^2+3x+1=0$

Câu 4.(2 điểm) Tìm giá trị nhỏ nhất của hàm số

$f(x)=\left | x-1 \right |+2\left | x-2 \right |+3\left | x-3 \right |+4\left | x-4 \right |+5\left | x-5 \right |$

Câu 5.(4 điểm) Cho tam giác ABC vuông tại A có AB=8cm,AC=6cm.Đường tròn tâm O nội tiếp tam giác tiếp xúc với hai cạnh AB,BC lần lượt tại E,F.Tia AO cắt EF  tại K.Chứng minh rằng tứ giác KFCO nội tiếp và tính diện tích tam giác OKC

Câu 6.(2 điểm) Cho tam giác ABC đều.Trên cạnh BC lấy điểm M sao cho $\widehat{BAM}=15^o$ .Đường thẳng qua điểm C và song song với đường thẳng AB cắt đường thẳng AM tại điểm N.Chứng minh rằng $\frac{1}{AM^2}+\frac{1}{AN^2}=\frac{3}{4AB^2}$




#501149 Cho điểm M cố đinh nằm ngoài đường tròn (O;R)

Đã gửi bởi Pham Le Yen Nhi on 24-05-2014 - 09:37 trong Hình học

Vì sao $\widehat{NOK}=\frac{1}{2}\widehat{AOB}$

tính chất 2 tiếp tuyến cắt nhau :))

$OH$ là phân giác góc $AOC$

$OI$ là phân giác góc $COB$




#479603 $A=\sqrt{x+4}+\sqrt{4-x}-\sqrt{16-x^{2}}$

Đã gửi bởi Pham Le Yen Nhi on 28-01-2014 - 11:26 trong Bất đẳng thức và cực trị

Đề là dấu trừ nhé bạn?? bạn giải giùm

mình nhìn nhầm đề  :(




#479597 $A=\sqrt{x+4}+\sqrt{4-x}-\sqrt{16-x^{2}}$

Đã gửi bởi Pham Le Yen Nhi on 28-01-2014 - 11:18 trong Bất đẳng thức và cực trị

Ta đặt : $\sqrt{x+4}=a$, $\sqrt{4-x}=b$

$\Rightarrow \sqrt{16-x^2}=ab$

Ta tìm max của biểu thức  $N = a+b+ab$

Ta có $(a+b)^2 \leq 2(a^2+b^2)= 2.8=16 => a+b\leq 4$

và $a^2+b^2 \geq 2ab =>ab \leq \frac{(a^2+b^2)}{2}=\frac{8}{2}=4$

$=> N\leq 4 +4 =8$

Dấu "=" xảy ra khi $x=0$




#542983 Đề thi chọn đội tuyển Olympic 30-4 THPT chuyên Lê Hồng Phong TP.HCM 2014-2015

Đã gửi bởi Pham Le Yen Nhi on 04-02-2015 - 19:39 trong Thi HSG cấp Tỉnh, Thành phố. Olympic 30-4. Đề thi và kiểm tra đội tuyển các cấp.

Bài 1: Giải phương trình:

$\sqrt{2x+15}=32x^{2}+32x-20$

Bài 2: $R^{+}$ là tập hợp các số thực dương. Tìm tất cả các hàm số $f$: $R^{+}\rightarrow R^{+}$ thỏa

$f(x)f(y)=f(xy)+\frac{1}{x}+\frac{1}{y} \forall x,y\epsilon R^{+}$

Bài 3: Trong mặt phẳng tọa độ cho parabol (P): $y=-x^{2}+4px-p+1$ với p là một số hữu tỷ. Gọi S là diện tích tam giác có 2 đỉnh là 2 giao điểm của parabol (P) với trục hoành và đỉnh thứ ba là đỉnh của parabol (P). Tìm tất cả các số hữu tỷ p để S là một số nguyên.

Bài 4: Cho $a,b,c$ là các số thực không âm thỏa mãn $(a+b+2c)(b+c+2a)(c+a+2b)=1$

Chứng minh rằng: 

$\frac{a}{b(4c+15)(b+2c)^{2}}\frac{b}{c(4a+15)(c+2a)^{2}}\frac{c}{a(4b+15)(a+2b)^{2}}\geq \frac{1}{3}$

Bài 5: Cho các số nguyên dương $k_{1}<k_{2}<...<k_{n}<k_{n+1}<...$, trong đó không có 2 số liên tiếp. Đặt $S_{n}=k_{1}+k_{2}+...+k_{n}$. Chứng minh rằng $\left [S_{n};S_{n+1} \right )$ có ít nhất một số chính phương với mọi n.

Bài 6 : Cho D là điểm nằm trên cạnh BC của tam giác ABC sao cho $\angle CAD=\angle CBA$. Một đường tròn tâm O qua hai điểm B,D cắt cạnh AB,AD lần lượt tại E,F. Đường thẳng BF và DE cắt nhau tại G. M là trung điểm AG. Chứng minh CM vuông góc với AO.

 




#500970 Cho điểm M cố đinh nằm ngoài đường tròn (O;R)

Đã gửi bởi Pham Le Yen Nhi on 23-05-2014 - 16:26 trong Hình học

Cho điểm M cố đinh nằm ngoài đường tròn (O;R),vẽ hai tiếp tuyến MA,MB với đường tròn (O;R)(A,B là các tiếp điểm).Trên cung nhỏ AB lấy một điểm C (C khác A và B). Vẽ tiếp tuyến qua C cắt MA tại H và MB tại I. Đường thẳng AB cắt OH,OI lần lượt tại N và K. chứng minh

a) $\widehat{OBN}=\widehat{OIN}$

b) 4 điểm N,H,I,K thuộc 1 đường tròn

c) Tỉ số $\frac{IH}{NK}=const$ khi C di chuyển trên cung nhỏ AB của đường tròn (O;R)

Chứng minh:

a) $\angle ABI =\angle NOK= \frac{1}{2}\angle AOB$ nên tứ giác $OBIN$nội tiếp $\Rightarrow \angle OBN = \angle OIN$

b) Cmtt $\Rightarrow OKAH$ là tứ giác nội tiếp

$\Rightarrow \angle HKI = \angle HNI= 90^{\circ}$

$\Rightarrow NKIH$ là tứ giác nội tiếp (đpcm)

c) Dễ thấy $\Delta ONK \sim \Delta OIH (g-g)$ 

$\Rightarrow \frac{IH}{NK}=\frac{OH}{OK}=\frac{1}{\frac{OK}{OH}}=\frac{1}{cos\angle NOK}= const$

(vì $M$ cố định nên $A,B$ cố định, $O$ cố định $\Rightarrow \angle AOB$ không đổi $\Rightarrow \angle NOK = \frac{1}{2}\angle OAB$ không đổi)

p/s: Bạn tự vẽ hình nhé :))




#528436 $\sqrt{2x-3}+\sqrt{5-2x}=3x^{2}-...

Đã gửi bởi Pham Le Yen Nhi on 12-10-2014 - 14:16 trong Phương trình, hệ phương trình và bất phương trình

Giải phương trình

a. $\sqrt{7-x}+\sqrt{x-5}= x^{2}-12x+38$

b.$\sqrt{2x-3}+\sqrt{5-2x}=3x^{2}-12x+4$

a) Ta có 

$VT^{2}=(\sqrt{7-x}+\sqrt{x-5})^{2}\leq 4\Rightarrow VT\leq 2$

 $VP=x^{2}-12x+38=(x-6)^{2}+2\geq 2$

Mà VT=VP.

Dấu "=" xảy ra $\Leftrightarrow x=6$

Thử lại x=6 thỏa.

Vậy nghiệm của pt đã cho là x=6.




#495597 Phương trình $x^2-(m-1)^2x+m=0$ có các nghiệm đều nguyên.

Đã gửi bởi Pham Le Yen Nhi on 27-04-2014 - 23:10 trong Phương trình, hệ phương trình và bất phương trình

Anh giải như thế này...chưa biết đúng hay sai nữa :

Điều kiện cần để phương trình có nghiệm nguyên là biệt thức là một chính phương :

$\Delta =(m-1)^{4}-4m=k^{2}(k\in \mathbb{Z})\\$

Đặt $y=m-1 ( y \in \mathbb{Z})$ khi đó :

$(m-1)^{4}-4m=y^{4}-4(y+1)=y^{4}-4y-4\\=(y^{2}-2)^{2}+(2y-1)^{2}-9=k^{2}\\ \Leftrightarrow (y^{2}-2)^{2}+(2y-1)^{2}=m^{2}+3^{2}\\ \Leftrightarrow \begin{bmatrix} \left\{\begin{matrix} y^{2}-2=m\\ 2y-1=3 \end{matrix}\right.\\ \left\{\begin{matrix} y^{2}-2=3\\ 2y-1=m \end{matrix}\right.\\ \end{bmatrix} \Leftrightarrow \begin{bmatrix} \left\{\begin{matrix} m=y^{2}-2=2\\ y=2(TM) \end{matrix}\right.\\ \left\{\begin{matrix} y^{2}=5(L)\\ m=2y-1 \end{matrix}\right.\\ \end{bmatrix}$

Suy ra $m=3$.Thử lại phương trình ban đâu ta thấy thỏa mãn.

Vậy $m=3$ là yêu cầu của bài toán.

-----------------------------------------------------------------------------------------------------------

sao em thay m=0 vào vẫn thỏa mãn bài toán mà @@




#493560 Tìm min, Max

Đã gửi bởi Pham Le Yen Nhi on 17-04-2014 - 20:08 trong Bất đẳng thức và cực trị

Chắc điều kiện là x,y>0

Ta có: 

$B=x^{2}y^{2}+\frac{1}{32x}+\frac{1}{32x}+\frac{1}{32y}+\frac{1}{32y}+\frac{15}{16}(\frac{1}{x}+\frac{1}{y})\geq 5\sqrt[5]{\frac{x^{2}y^{2}}{2^{20}x^{2}y^{2}}}+\frac{15.4}{16(x+y)}=2\frac{5}{16}+\frac{15}{4}=\frac{65}{16}$

Nếu x,y >0 thì mình còn 1 cách giải khác :))

$B= x^{2}y^{2}+\frac{1}{x}+\frac{1}{y}= \frac{1}{64xy}+\frac{1}{64xy}+x^{2}y^{2}+\frac{31}{32xy}\geq 3\sqrt[3]{\frac{1}{64xy}.\frac{1}{64xy}.x^{2}y^{2}} + \frac{31.4}{32(x+y)^{2}}$

$\Rightarrow B\geq \frac{3}{16}+\frac{31}{8}=\frac{65}{16}$

Dấu "=" xảy ra $\Leftrightarrow x=y=\frac{1}{2}$ 




#493545 Tìm min, Max

Đã gửi bởi Pham Le Yen Nhi on 17-04-2014 - 19:50 trong Bất đẳng thức và cực trị

Bài 1: Cho $x, y \geq 0$ ; $x + y = 1$

Tìm min, Max của $P = \frac{x}{y + 1} + \frac{y}{x + 1}$

 

 

Ta có 

 

$P=\frac{x}{y+1}+\frac{y}{x+1}=\frac{x^{2}+x+y^{2}+y}{xy+1+x+y}=\frac{x^{2}+y^{2}+1}{xy+2}=\frac{(x+y)^{2}-2xy+1}{xy+2}$

$\Rightarrow P=\frac{2-2xy}{xy+2}$

Có: $xy \leq \frac{1}{4}$

Đặt $xy=t$ thì $0\leq t\leq \frac{1}{4}$

Khi đó $P = \frac{2-2t}{t+2}=-2 +\frac{6}{t+2}$

$min P \Leftrightarrow min\frac{6}{t+2} \Leftrightarrow t=\frac{1}{4}\Leftrightarrow x=y=\frac{1}{2}\Rightarrow min P = \frac{2}{3}$

Tương tự tìm được $maxP = 1 \Leftrightarrow$ một trong 2 số $x,y$ có một số bằng 0, một số bằng 1




#631794 Đề thi học sinh giỏi môn toán khối 11 khu vực DUYÊN HẢI VÀ ĐỒNG BẰNG BẮC BỘ n...

Đã gửi bởi Pham Le Yen Nhi on 07-05-2016 - 20:17 trong Thi HSG cấp Tỉnh, Thành phố. Olympic 30-4. Đề thi và kiểm tra đội tuyển các cấp.

Câu 1:

 

Ta có:

$(n+3)u_{n+2}=2(n+2)^{2}u_{n+1}-(n+1)^{2}(n+2)u_{n}$

$\Leftrightarrow (n+3)u_{n+2}-(n+2)^{2}u_{n+1}=(n+2)((n+2)u_{n+1}-(n+1)^{2}u_{n})$

Đặt:

$x_{n}=(n+1)u_{n}-n^{2}u_{n-1}$

$\Rightarrow x_{n}=nx_{n-1}=n(n-1)x_{n-2}=...=\frac{2017n!}{2}$

Từ đó ta có:

$(n+1)u_{n}-n^{2}u_{n-1}=\frac{2017n!}{2}$

Từ đây tính được $u_{n}=\frac{1}{n+1}(n!+\frac{(n-1)2017n!)}{2})$




#554899 Chứng minh rằng $\frac{HP}{HQ}=\frac{...

Đã gửi bởi Pham Le Yen Nhi on 18-04-2015 - 20:20 trong Hình học phẳng

Bài 1: Cho tam giác $ABC$ trực tâm $H$. Một đường thẳng bất kì qua H cắt AB,AC tại P,Q. Đường thẳng qua $H$ vuông góc $PQ$ cắt $BC$ tại $M$. Chứng minh rằng $\frac{HP}{HQ}=\frac{MB}{MC}$

Bài 2: Cho tứ giác $ABCD$ nội tiếp (O). $M,N$ là trung điểm $AB,CD$. Đường tròn ngoại tiếp tam giác $ANB$ cắt $CD$ tại $Q$, đường tròn ngoại tiếp tam giác $MCD$ cắt $AB$ tai $P$. Chứng minh rằng $AC,BD,PQ$ đồng quy.




#493416 Cho tam giác $ABC$ cân tại $A$ ngoại tiếp $(O)$...

Đã gửi bởi Pham Le Yen Nhi on 16-04-2014 - 21:50 trong Hình học

Làm sao ra được như vậy

Đề thực ra thế này:
 

 

(D;E;F thuộc AB;BC;CA);BF cắt đường tròn tại I; DI cắt BC tại M.

1. Cm: $DEF$ có 3 góc nhọn

2. Cm: $DF//BC$
3. Cm: $BDFC$ nội tiếp

4. Cm: $BD.CF=BM.BC$

 

c) Dễ thấy $\angle ADF =\angle ABC =\angle BCA$ ( do $\Delta ABC$ cân tại $A$)

nên tứ giác $BDFC$ nội tiếp

d) Ta có $\angle BDM =\angle DFI =\angle FBC$  và $\angle BDM =\angle FCB$ 

Nên $\Delta DBM \sim \Delta BCF$ (g-g)

$\Rightarrow \frac{BD}{BC}= \frac{BM}{CF}\Rightarrow BD.CF=BC.BM$ (ĐPCM)

p/s: mình không kịp vẽ hình nha :))




#495704 Cho a, b, c là các số nguyên dương. So sánh: $\frac{a}...

Đã gửi bởi Pham Le Yen Nhi on 28-04-2014 - 18:24 trong Bất đẳng thức và cực trị

Cho a, b, c là các số nguyên dương. So sánh:

$\frac{a}{a+b}+\frac{b}{b+c}+\frac{c}{c+a}$ và 1

ta có a,b,c >0 nên

$\frac{a}{a+b}>\frac{a}{a+b+c}$

$\frac{b}{b+c}>\frac{b}{a+b+c}$

$\frac{c}{a+c}>\frac{a}{a+b+c}$

$\Rightarrow \frac{a}{a+b}+\frac{b}{b+c}+\frac{c}{c+a}> \frac{a+b+c}{a+b+c}=1$




#496625 Tìm vị trì điểm D để DM lớn nhất?

Đã gửi bởi Pham Le Yen Nhi on 02-05-2014 - 18:18 trong Hình học

Giải dùm mình câu c với?

Cho tam giác ABC đều, nội tiếp đường tròn tâm O, bán kính R. Điểm D nằm trên cung BC, AD cắt BC tại M

a) Cm: DB+DC=AD

b) Cm: AD.AM không đổi

a) Gọi $K$ là một điểm thuộc $AD$ sao cho $KD=BD$

Dễ thấy $\Delta BDK$ đều và chứng minh được $\Delta AKB = \Delta CDB \Rightarrow AK=CD$

Khi đó $AD=AK+KD=BD+DC$ (đpcm)

b) Ta đặt $AB=AC=BC=a$

Có $AD.AM =(DB+CD).AM = DB.AM+CD.AM= MC.AB+BM.AC= (MB+MC)a=a^{2}$ không đổi (đpcm)

P/s: Bạn tự vẽ hình nha :))




#487003 CMR: trung trực DE đi qua điểm cố định khi M di động

Đã gửi bởi Pham Le Yen Nhi on 15-03-2014 - 19:53 trong Hình học

DE đi qua tâm đường tròn ngoại tiếp tam giác ABC thì phải  :mellow: